Загрузка страницы

Solving the Schrödinger Equation for a free particle

In this video I will Solve the Schrödinger Equation for a free particle and find the expresion for psi(x,t).

In the next video we will begin doing a few problems to practice.

My name is Nick Heumann, I am a recently graduated physicist. I love to teach physics, so I decided to give YouTube a try. English is not my first language, but I hope that you can understand me well enough regardless.
▬ Contents of this video ▬▬▬▬▬▬▬▬▬▬
00:00 Start

Видео Solving the Schrödinger Equation for a free particle канала Nick Heumann
Показать
Комментарии отсутствуют
Введите заголовок:

Введите адрес ссылки:

Введите адрес видео с YouTube:

Зарегистрируйтесь или войдите с
Информация о видео
28 января 2021 г. 5:00:34
00:20:27
Другие видео канала
Griffiths Problem 4.4 (3rd ed) Solution: The OTHER Solution to the Angular EquationGriffiths Problem 4.4 (3rd ed) Solution: The OTHER Solution to the Angular EquationGriffiths QM Problem 4.8: Infinite Spherical Well for l=1Griffiths QM Problem 4.8: Infinite Spherical Well for l=1The EASIEST Explanation of the Gram-Schmidt Procedure (Griffiths QM 3rd ed. Problem 3.4)The EASIEST Explanation of the Gram-Schmidt Procedure (Griffiths QM 3rd ed. Problem 3.4)Griffiths Intro to QM - Problem 8.15: The ULTIMATE WKB Problem -HARDEST ONE YETGriffiths Intro to QM - Problem 8.15: The ULTIMATE WKB Problem -HARDEST ONE YETProving orthogonality and Completeness of Infinite Square Well WavefunctionsProving orthogonality and Completeness of Infinite Square Well WavefunctionsThe Integral You HAVE TO KNOW to MASTER Quantum Mechanics!The Integral You HAVE TO KNOW to MASTER Quantum Mechanics!How do LASERs work? A Quantum Mechanical Explanation Using Perturbation TheoryHow do LASERs work? A Quantum Mechanical Explanation Using Perturbation TheoryHow to determine the 4 Wavefunctions of the Hydrogen Atom for n=2How to determine the 4 Wavefunctions of the Hydrogen Atom for n=2How To Go From Arcsin(x) to Ln (natural logarithm) (Arcsin - Log Formulation)How To Go From Arcsin(x) to Ln (natural logarithm) (Arcsin - Log Formulation)Griffiths QM 6.26 (3rd ed) 6.28 (2nd ed): Hyperfine Splitting for Muonic H, Positronium, MuoniumGriffiths QM 6.26 (3rd ed) 6.28 (2nd ed): Hyperfine Splitting for Muonic H, Positronium, MuoniumGriffiths QM 2.27 Solution: Finding Transmission coefficient for double delta potentialGriffiths QM 2.27 Solution: Finding Transmission coefficient for double delta potentialKlein-Gordon and Feynman Propagators,Time Ordering (Peskin & Schroeders Eq. 2.54 and 2.56 EXPLAINED)Klein-Gordon and Feynman Propagators,Time Ordering (Peskin & Schroeders Eq. 2.54 and 2.56 EXPLAINED)The WKB Approximation for Energy less than V - Finding the Transmission coefficientThe WKB Approximation for Energy less than V - Finding the Transmission coefficientWhy we need QFT & Derivation of Klein-Gordon Langriangian DensityWhy we need QFT & Derivation of Klein-Gordon Langriangian DensityHow to Solve The Infinite Spherical Well (FULLY EXPLAINED)How to Solve The Infinite Spherical Well (FULLY EXPLAINED)Griffiths QM 6.13 (3d edition) Solution: Relativistic Correction for Quantum Harmonic OscillatorGriffiths QM 6.13 (3d edition) Solution: Relativistic Correction for Quantum Harmonic OscillatorWKB Approximation Connection Formulas Example: Potential Well with no Vertical WallsWKB Approximation Connection Formulas Example: Potential Well with no Vertical WallsVariational Principle in QM Example 1: Quantum Harmonic OscillatorVariational Principle in QM Example 1: Quantum Harmonic OscillatorGriffiths QM 4.15 Solution: Proving Radial Wavefunction and finding expectation values for l=n-1Griffiths QM 4.15 Solution: Proving Radial Wavefunction and finding expectation values for l=n-1Solving the Schrödinger Equation for the Infinite Potential Well (Particle in a square box)Solving the Schrödinger Equation for the Infinite Potential Well (Particle in a square box)Griffiths QM Problem 2.37 (3rd edition): Find expectation values for nth stationary state of QHOGriffiths QM Problem 2.37 (3rd edition): Find expectation values for nth stationary state of QHO
Яндекс.Метрика